S 02 Part 2-2i1tr7g
S 02 Part 2-2i1tr7g
Printed Name:
Signature:
PHYSICS DEPARTMENT
UNIVERSITY OF OREGON
Ph.D. Qualifying Examination, PART II
Tuesday, April 2, 2002, 1:00 p.m. to 5:00 p.m.
The examination papers are numbered in the upper right-hand corner of each page.
Print and then sign your name in the spaces provided on this page. For identication
purposes, be sure to submit this page together with your answers when the exam is
nished. Be sure to place both the exam number and the question number on any
additional pages you wish to have graded.
There are six equally weighted questions, each beginning on a new page. Read all six
questions before attempting any answers.
Begin each answer on the same page as the question, but continue on additional blank
pages if necessary. Write only on one side of each page. Each page should contain work
related to only one problem. If you need extra space for another problem, start a new page.
If you need to leave your seat, wait until everyone else is seated before approaching the
proctor.
Calculators may be used only for arithmetic. Calculators with stored equations
or text are not allowed. Dictionaries may be used if they have been approved by the
proctor before the examination begins. No other papers or books may be used.
When you have nished, come to the front of the room and hand your examination
paper to the proctor; rst put all problems in numerical order and staple them together.
Please make sure you follow all instructions carefully. If you fail to follow instructions,
or to hand your exam paper in on time, an appropriate number of points may be
subtracted from your nal score.
Constants
1.60 1019 C
9.11 1031 kg (0.511 MeV/c2 )
1.673 1027 kg (938 MeV/c2 )
1.675 1027 kg (940 MeV/c2 )
80.4 GeV/c2
6.63 1034 J s
3.00 108 m/s
1.38 1023 J/K
6.67 1011 N m2/kg2
4 107 H/m
8.85 1012 F/m
5.98 1024 kg
6.38 106 m
6.96 108 m
5.8 103 K
1.50 1011 m
7.88 103 kg/m3
2.82 1015 m
9.8 m/s2
1.7 1027 kg
21.1 J/mole K
29.4 J/mole K
Stirlings Formula
ln(x!) = x ln(x) x ln
2x + O(1/x)
Integrals
dx x e
0
2n ax2
1 3 5 (2n 1)
=
2n an
dx n x
= (n)
x e
x
Problem 1
Consider a mechanical system described by a single coordinate x(t) that evolves
according to the Lagrangian
1
x 2
L= m
.
2 1 (x/a)2
a) Find the Euler-Lagrange equation of motion for x(t).
b) The system begins at t = 0 with initial conditions x(0) = a/2, x(0)
= 0. What will be
its subsequent behavior?
c) [This part slightly revised from the original. Ed.] Now suppose that the system begins at
t = 0 with initial conditions x(0) = 0, x(0)
Problem 2
Two particles, each having the same mass m, move in the {x, y} plane, where y is the
vertical direction. The particles are joined by a rigid, massless rod of length L. The center
of the rod is connected by a frictionless pivot to the end of a second rigid, massless rod,
which has a length r. The top end of the second rod is connected to a xed support
structure by a frictionless pivot. The system can be characterized by the two angles and
shown in the gure.
a) Write down the Lagrangian for this system and use it to write the Euler-Lagrange
equations that determine the time evolution of (t) and (t).
b) Find the general solution of these equations for which (t) 0.
c) Qualitatively discuss the nature of the motion for which (t) is not identically equal to
zero.
Problem 3
The earth is continually bombarded by matter dust and meteors from space. This
bombardment has a small eect on the earths rotation.
a) Show that the moment of inertia of a thin spherical shell of mass m and radius R is
(2/3) mR2 .
b) Show that the moment of inertia of a solid sphere of mass M and radius R is (2/5) M R2 .
c) Suppose that in a given year a mass m of material falls on earth randomly from all
directions. Calculate the fractional change in the length of the day per year in terms of m
and the mass M of the earth.
Problem 4
Consider a cylindrically symmetric current density given by
jx (x) = 0
jy (x) = 0
jz (x) = j(r),
where r =
x2 + y 2 .
j(r) =
I0 /(r02 ) for r r0
,
0
for r > r0
for r = r0 .
Problem 5
A particle of charge q and mass m is released at rest from the origin in the presence of
are
a uniform electric eld E = E0 k and a uniform magnetic eld B = B0j, where {i, j, k}
unit vectors along the x, y, and z axes respectively.
a) Qualitatively explain the motion of the particle after it has been released, and sketch the
expected trajectory.
b) An elegant relativistic solution can be obtained by performing a Lorentz transformation
from the original frame S into a frame S where the electric eld vanishes. Find the boost
needed to make E = 0. (Assume E0 < B0 .)
Hint: If a four-vector transforms as p = p, then the eld tensor transforms as
F = F T , where
0
Ex /c Ey /c Ez /c
E /c
0
Bz By
x
F =
,
Ey /c Bz
0
Bx
Ez /c By Bx
0
and () is the Lorentz transformation matrix. (If you wish, you may work in units where
c = 1.)
c) Solve for the trajectory of the particle in the frame S . Do not assume non-relativistic
motion, although you may assume that the particle does not radiate (ie., ignore radiation
damping). Explain how this solution compares to what you described in part a).
Problem 6
Two mutually coherent, monochromatic, plane light waves with equal electric eld
amplitudes, but unequal angular frequencies 1 and 2 , co-propagate in free space in the
z-direction. Assume |2 1 | is small compared to 1 or 2 . Wave 1 is linearly polarized
in the x-direction and wave 2 is right-circularly polarized. (Mutually coherent waves can
be formed, for example, by splitting a beam into two by use of a half-silvered mirror and
frequency shifting one of them by means of the Doppler eect.)
a) Describe quantitatively the intensity of the light as a function of time striking a fast
detector placed with its surface perpendicular to the z-axis. (The detectors response time
is less than |2 1 |1 .)
b) Describe qualitatively the behavior of the intensity if one eld is monochromatic but the
other has a spectral width that equals roughly three-fourths of the frequency dierence
|2 1 |. (This could result from one eld traveling through a uctuating medium.)